Difference between revisions of "2011 AMC 10B Problems/Problem 20"
Bunnypower (talk | contribs) (→Problem) |
(First edit - any feedback appreciated!) |
||
(6 intermediate revisions by 5 users not shown) | |||
Line 34: | Line 34: | ||
== Solution 3== | == Solution 3== | ||
− | We split rhombus <math>ABCD</math> into two equilateral triangles, <math>ABD</math> and <math>BCD</math>. In triangle <math>ABD</math>, the probability that a randomly selected point is closer to <math>B</math> than either other point is | + | We split rhombus <math>ABCD</math> into two equilateral triangles, <math>ABD</math> and <math>BCD</math>. In triangle <math>ABD</math>, the probability that a randomly selected point is closer to <math>B</math> than either other point is <math>\frac{1}{3}</math> *. Similarly, in triangle <math>BCD</math>, the same principle applies. Thus, the area of the region closer to <math>B</math> than <math>A</math>, <math>C</math>, or <math>D</math> is <math>\frac{1}{3} [ABD] + \frac{1}{3} [BCD]</math>. Since <math>ABD</math> and <math>BCD</math> are congruent, we have <math>\frac{1}{3} [ABD] + \frac{1}{3} [BCD] = \frac{2}{3} [ABD] = \frac{2}{3} \cdot \frac{s^2\sqrt{3}}{4} = \frac{2}{3} \cdot \frac{(2)^2\sqrt{3}}{4} = \boxed{\frac{2\sqrt3}{3} = C}</math>, and we are done. |
+ | |||
+ | \*See note | ||
+ | |||
+ | == Solution 4 == | ||
+ | <asy> | ||
+ | unitsize(8mm); | ||
+ | defaultpen(linewidth(0.8pt)+fontsize(10pt)); | ||
+ | dotfactor=4; | ||
+ | |||
+ | pair A=(4,0), B=(2,2sqrt(3)), C=(-2,2sqrt(3)), D=(0,0), E=(B+C)/2, F=(B+C+D)/3, G=(A+B+D)/3, H=(A+B)/2; | ||
+ | fill((0,2sqrt(3))--B--(3,sqrt(3))--(2,(2sqrt(3))/3)--(0,(4sqrt(3))/3)--cycle,gray); | ||
+ | draw(A--B--C--D--cycle); | ||
+ | draw(D--(0,2sqrt(3))); draw(D--(3,sqrt(3))); draw(A--C); draw(F--B--H); draw(B--G); | ||
+ | |||
+ | label("$A$",A,SE);label("$B$",B,NE);label("$C$",C,NW);label("$D$",D,SW); | ||
+ | label("$E$",E,N);label("$F$",F,SW);label("$G$",G,S);label("$H$",H,E); | ||
+ | label("$2$",(D--C),SW); | ||
+ | </asy> | ||
+ | Since <math>H</math> and <math>E</math> are halfway between <math>AB</math> and <math>CB</math>, respectively, we know that <math>\overline{BH}=\overline{BE}=1</math>. By symmetry, <math>\Delta BFG</math> is equilateral, so <math>\angle FBG=60^\circ\implies\angle EBF=\angle HBG=30^\circ</math> and therefore <math>\Delta EBF</math> and <math>\Delta HBG</math> are 30-60-90 right triangles. | ||
+ | Thus, <math>[\Delta EBF]=[\Delta BFG]=\dfrac1{2\sqrt3}</math>. | ||
+ | We know that <math>\overline{FB}=\overline{GB}=\dfrac2{\sqrt3}</math>, so therefore <math>[\Delta BFG]=\dfrac{\sqrt3}4\left(\dfrac2{\sqrt3}\right)^2=\dfrac1{\sqrt3}</math>. | ||
+ | Summing these three regions, we get <math>\dfrac1{2\sqrt3}+\dfrac1{2\sqrt3}+\dfrac1{\sqrt3}=\boxed{\textbf{(C)}~\dfrac{2\sqrt3}3}</math>. | ||
+ | ~ Technodoggo, Asymptote diagram modified from Solution 1 | ||
+ | |||
+ | == Solution 5 == | ||
+ | <asy> | ||
+ | unitsize(8mm); | ||
+ | defaultpen(linewidth(0.8pt)+fontsize(10pt)); | ||
+ | dotfactor=4; | ||
+ | |||
+ | pair A=(4,0), B=(2,2sqrt(3)), C=(-2,2sqrt(3)), D=(0,0), E=(B+C)/2, F=(B+C+D)/3, G=(A+B+D)/3, H=(A+B)/2; | ||
+ | fill((0,2sqrt(3))--B--(3,sqrt(3))--(2,(2sqrt(3))/3)--(0,(4sqrt(3))/3)--cycle,gray); | ||
+ | draw(A--B--C--D--cycle); | ||
+ | draw(D--F); draw(D--G); draw(F--C); draw(A--G); draw(F--B); draw(B--G); | ||
+ | draw(B--D); | ||
+ | |||
+ | label("$A$",A,SE);label("$B$",B,NE);label("$C$",C,NW);label("$D$",D,SW); | ||
+ | label("$E$",E,N);label("$F$",F,SW);label("$G$",G,S);label("$H$",H,E); | ||
+ | label("$2$",(D--C),SW); | ||
+ | </asy> | ||
+ | To keep it simple, break rhombus <math>ABCD</math> into two triangles, <math>ABD</math> and <math>BCD</math>. To see the area closest to the point <math>B</math>, notice that a third of each triangle, which contains all the points nearest to <math>B</math> in each triangle, is easily visualizable. Thus, a third of rhombus <math>ABCD</math> must be found. | ||
+ | |||
+ | We find the total area of rhombus <math>ABCD</math>, which we can again split into two congruent equilateral triangles with side length <math>2</math>. Using the formula of equilateral triangles and then multiplying by <math>\dfrac{1}{3}</math>: | ||
+ | <cmath>\dfrac{\sqrt{3}}{4}\cdot 2^2 \cdot 2 \cdot \dfrac{1}{3} = 2\sqrt{3} \cdot \dfrac{1}{3} = \boxed{\textbf{(C)}~\dfrac{2\sqrt{3}}{3}}</cmath> | ||
+ | -NSAoPS, diagram modified from Solution 1. | ||
+ | |||
+ | ==Note== | ||
+ | There is an intuitive explanation for why the shaded region is one third of the rhombus. Consider one of the equilateral triangles; the figure is symmetrical, so one half of the shaded region should be in the triangle. Note that the perpendicular bisectors we constructed in previous solutions to deliminate <math>R</math> must be medians, as the triangle is equilateral Therefore, recalling that the three intersecting medians of a triangle divide it into 6 smaller triangles of equal area, we observe that the shaded region in the equilateral triangles includes precisely two of these smaller triangles. We can repeat this with the other equilateral triangle that forms the rhombus, and thus we conclude that the shaded area is one third of the rhombus's total area. | ||
+ | |||
+ | ~LeonQS | ||
+ | |||
+ | ==Video Solution by TheBeautyofMath== | ||
+ | https://youtu.be/gCmQlaiEG5A | ||
+ | |||
+ | ~IceMatrix | ||
== See Also== | == See Also== |
Latest revision as of 22:18, 14 August 2025
- The following problem is from both the 2011 AMC 12B #16 and 2011 AMC 10B #20, so both problems redirect to this page.
Contents
Problem
Rhombus has side length
and
°. Region
consists of all points inside the rhombus that are closer to vertex
than any of the other three vertices. What is the area of
?
Solution
Suppose that is a point in the rhombus
and let
be the perpendicular bisector of
. Then
if and only if
is on the same side of
as
. The line
divides the plane into two half-planes; let
be the half-plane containing
. Let us define similarly
and
. Then
is equal to
. The region turns out to be an irregular pentagon. We can make it easier to find the area of this region by dividing it into four triangles:
Since
and
are equilateral,
contains
,
contains
and
, and
contains
. Then
with
and
so
. Multiply this by 4 and it turns out that the pentagon has area
.
Solution 2
We follow the steps shown above until we draw pentagon . We know that rhombus
can be divided into equilateral triangles
and
. Using the
special right triangle rules, we find the height of the equilateral triangles (and the height of the rhombus) to be
. Therefore, the area of
is
. We now have to take off the areas
,
, and
to get the desired shape.
is just half of
and
and
are each
, for a total area of
.
Solution 3
We split rhombus into two equilateral triangles,
and
. In triangle
, the probability that a randomly selected point is closer to
than either other point is
*. Similarly, in triangle
, the same principle applies. Thus, the area of the region closer to
than
,
, or
is
. Since
and
are congruent, we have
, and we are done.
\*See note
Solution 4
Since
and
are halfway between
and
, respectively, we know that
. By symmetry,
is equilateral, so
and therefore
and
are 30-60-90 right triangles.
Thus,
.
We know that
, so therefore
.
Summing these three regions, we get
.
~ Technodoggo, Asymptote diagram modified from Solution 1
Solution 5
To keep it simple, break rhombus
into two triangles,
and
. To see the area closest to the point
, notice that a third of each triangle, which contains all the points nearest to
in each triangle, is easily visualizable. Thus, a third of rhombus
must be found.
We find the total area of rhombus , which we can again split into two congruent equilateral triangles with side length
. Using the formula of equilateral triangles and then multiplying by
:
-NSAoPS, diagram modified from Solution 1.
Note
There is an intuitive explanation for why the shaded region is one third of the rhombus. Consider one of the equilateral triangles; the figure is symmetrical, so one half of the shaded region should be in the triangle. Note that the perpendicular bisectors we constructed in previous solutions to deliminate must be medians, as the triangle is equilateral Therefore, recalling that the three intersecting medians of a triangle divide it into 6 smaller triangles of equal area, we observe that the shaded region in the equilateral triangles includes precisely two of these smaller triangles. We can repeat this with the other equilateral triangle that forms the rhombus, and thus we conclude that the shaded area is one third of the rhombus's total area.
~LeonQS
Video Solution by TheBeautyofMath
~IceMatrix
See Also
2011 AMC 10B (Problems • Answer Key • Resources) | ||
Preceded by Problem 19 |
Followed by Problem 21 | |
1 • 2 • 3 • 4 • 5 • 6 • 7 • 8 • 9 • 10 • 11 • 12 • 13 • 14 • 15 • 16 • 17 • 18 • 19 • 20 • 21 • 22 • 23 • 24 • 25 | ||
All AMC 10 Problems and Solutions |
2011 AMC 12B (Problems • Answer Key • Resources) | |
Preceded by Problem 15 |
Followed by Problem 17 |
1 • 2 • 3 • 4 • 5 • 6 • 7 • 8 • 9 • 10 • 11 • 12 • 13 • 14 • 15 • 16 • 17 • 18 • 19 • 20 • 21 • 22 • 23 • 24 • 25 | |
All AMC 12 Problems and Solutions |
These problems are copyrighted © by the Mathematical Association of America, as part of the American Mathematics Competitions.